Sei sulla pagina 1di 15

A questionable aspect of the reasoning above is that it Aiesha responds to Adam's argument by

All of the following weakens the politician's argument EXCEPT?


Clarification of which one of the following issues would be most important to an evaluation of the skeptics' position? Each of the following describes a flaw in the psychologist's reasoning EXCEPT Each of the following precepts is logically consistent with the columnist's conclusion EXCEPT Each of the following principles is logically consistent with the columnist's conclusion EXCEPT Each of the following, if true, strengthens the toxicologist's argument EXCEPT

Flaw

Method

Weaken X

Evaluate

Flaw X

Cannot PR

Cannot PR

Strengthen X

Each of the following, if true, supports the claim above EXCEPT Each of the following, if true, supports the claim above EXCEPT:

Strengthen X

Strengthen X

Each of the following, if true, weakens the argument EXCEPT


Each of the following, if true, would strengthen the argument EXCEPT
If all of the statements above are true, which one of the following must be true? If the information above is correct, which one of the following conclusions can be properly drawn on the basis of it? If the information above is correct, which one of the following conclusions can be properly drawn? If the statements above are true, each of the following could be true EXCEPT

Weaken X

Strengthen X

Must Be True

Must Be True

Must Be True

Cannot Be True

If the statements above are true, which one of the following CANNOT be true?

Cannot Be True

If the statements above are true, which one of the following must also be true?

Must Be True

If the statements above are true, which one of the following must be false?

Cannot Be True

Larew and Mendota disagree about whether

Point at Issue

Of the following, which one most accurately expresses the main point of the argument? On the basis of their sentiment, Logan and Mendez are committed to disagreeing over whether On which one of the following assumptions does the argument rely?

Main Point

Point at Issue

Assumption

Ruth responds to Jorge's criticism by Sue challenges Anne's reasoning by The advertisement proceeds by
The answer to which one of the following questions would contribute most to an evaluation of the argument? The answer to which one of the following questions would most help in evaluating the columnist's argument?

Method

Method

Method

Evaluate

Evaluate

The argument assumes which one of the following? The argument derives its conclusion by

Assumption

Method

The argument employs which one of the following reasoning techniques?

Method

The argument is flawed because it The argument proceeds by


The claim that humans are still biologically adapted to a diet of wild foods plays which one of the following roles in the nutritionist's argument?

Flaw

Method

Method

The claim that people have positive or negative responses to many nonsense words plays which one of the following roles in the argument?

Method

The conclusion above follows logically if which one of the following is assumed?

Justify

The conclusion above is properly drawn if which one of the following is assumed?

Justify

The conclusion cited does not follow unless


The conclusion in the passage above relies on which of the following assumptions? The conclusion in the passage above relies on which one of the following assumptions?

Assumption

Assumption

Assumption

The dialogue above lends the most support to claim that Sherrie and Fran disagree with each other about which one of the following statements?

Point at Issue

The dialogue most supports the claim that Tony and Raoul disagree about whether The educators' reasoning provides grounds for accepting which of the following statements? The environmentalist's conclusion would be properly drawn if it were true that the The flawed pattern of reasoning in the argument above is most similar to that in which one of the following? The flawed reasoning in which one of the following is most similar to that in the commentator's argument? The information above most closely conforms to which one of the following principles? The information above provides the LEAST support for which one of the following?

Point at Issue

Must Be True

Justify

Parallel Flaw

Parallel Flaw

Strengthen PR

Must Be True X

The main point of the argument is that The method of the argument is to

Main Point

Method

The pattern of reasoning displayed in the argument above is most closely paralleled by that in which one of the following arguments?

Parallel Flaw

The phrase, "certain traits like herding ability risk being lost among pedigreed dogs" serves which one of the following functions in the argument?

Method

The position taken above presupposes which one of the following?

Assumption

The principle above conforms most to which of the following?


The principle above, if established, would justify which one of the following judgments? The questionable reasoning in the argument above is most closely paralleled by that in which one of the following?
The reasoning above is flawed because it fails to recognize that

Strengthen PR

Must PR

Parallel Flaw

Flaw

The reasoning in the argument is fallacious because the argument

Flaw

The reasoning in the argument is most vulnerable to criticism on the ground that the argument The reasoning in the argument is most vulnerable to criticism on the grounds that the argument

Flaw

Flaw

The reasoning in the astronomer's argument is flawed because this argument The statements above, if true, most strongly support which of the following? The statements above, if true, most strongly support which one of the following? The structure of the reasoning in the argument above is most parallel to that in which one of the following? Which one fo the following arguments exhibits flawed reasoning most similar to that exhibited by the argument above?
Which one of the following arguments has a flawed pattern of reasoning most like the flawed reasoning in the argument above?

Flaw

Must Be True

Must Be True

Parallel

Parallel Flaw

Parallel Flaw

Which one of the following arguments is most similar in its logical features to the argument above? Which one of the following arguments is most similar in its pattern of reasoning to the argument above?

Parallel

Parallel

Which one of the following can be inferred from the statement above? Which one of the following can be logically inferred from the passage?

Must Be True

Must Be True

Which one of the following can be properly inferred from Rosen's statement? Which one of the following can be properly inferred from the passage? Which one of the following conclusions is most strongly supported by the information above? Which one of the following describes the technique of reasoning used above?

Must Be True

Must Be True

Must Be True

Method

Which one of the following discoveries, if it were made, would most support the above hypothesis about South America and Africa?

Strengthen

Which one of the following exhibits a flawed pattern of reasoning most similar to that in the argument above? Which one of the following exhibits a pattern of reasoning most similar to that exhibited by the argument above? Which one of the following exhibits both of the logical flaws exhibited by the argument above? Which one of the following inferences is most strongly supported by the information above? Which one of the following inferences is most supported by the information above?

Parallel Flaw

Parallel

Parallel Flaw

Must Be True

Must Be True

Which one of the following is an argumentative strategy employed in the argument? Which one of the following is an assumption on which Barnes's argument depends? Which one of the following is an assumption on which the columnist's argument depends? Which one of the following is an assumption required by the argument above?
Which one of the following is an assumption that the art historian's argument requires in order for its conclusion to be properly drawn?

Method

Assumption

Assumption

Assumption

Assumption

Which one of the following is an assumption that would serve to justify the conclusion above? Which one of the following is an assumption upon which the argument depends? Which one of the following is most closely parallel in its reasoning to the reasoning in the argument above? Which one of the following is most strongly supported by the information above? Which one of the following is most strongly supported by the statements above, if they are true?

Justify

Assumption

Parallel

Must Be True

Must Be True

Which one of the following judgments best illustrates the principle illustrated by the argument above? Which one of the following judgments most closely conforms to the principle above?

Must PR

Must PR

Which one of the following logically follows from the statements above?
Which one of the following most accurately conforms to the principle above? Which one of the following most accurately describes a flaw in the argument's reasoning?
Which one of the following most accurately describes the relationship between Jane's argument and Mark's argument?
Which one of the following most accurately describes the role played in the argument by the statement that zooplankton feed upon phytoplankton?

Must Be True

Must PR

Flaw

Method

Method

Which one of the following most accurately describes the role played in the psychologist's argument by the claim that the obligation to express gratitude cannot be fulfilled anonymously?

Method

Which one of the following most accurately expresses the conclusion of the argument? Which one of the following most accurately expresses the conclusion of the journalist's argument?

Main Point

Main Point

Which one of the following most accurately expresses the main conclusion of the argument? Which one of the following most accurately expresses the point at issue between Ted and Mary? Which one of the following most accurately expresses the principle underlying the argumentation above? Which one of the following most accurately represents what is at issue between Jorge and Ruth? Which one of the following most accurately restates the main point of the passage? Which one of the following most logically completes the argument?

Main Point

Point at Issue

Justify PR

Point at Issue

Main Point

Main Point

Which one of the following most logically completes the passage


Which one of the following principles most helps to justify the reasoning above?

Main Point (fill in the blank)

Strengthen PR

Which one of the following propositions most helps to justify the reasoning above? Which one of the following statements, if true, most weakens the speaker's argument?

Strengthen PR

Weaken

Which one of the following would be most important to know in evaluating the hypothesis in the passage?
Which one of the following would it be most helpful to know in order to judge whether what the scientist subsequently learned calls into question the hypothesis?

Evaluate

Evaluate

Which one of the following would it be most relevant to investigate in evaluating the conclusion of George's argument?
Which one of the following, if all of them are true is LEAST helpful in establishing that the conclusion above is properly drawn?

Evaluate

Strengthen X

Which one of the following, if assumed, allows the conclusion above to be properly drawn? Which one of the following, if assumed, enables the argument's conclusion to be properly inferred? Which one of the following, if assumed, would allow the conclusion to be properly drawn? Which one of the following, if shown to be a realistic possibility, would undermine the argument? Which one of the following, if true, casts the most doubt on the conclusion above? Which one of the following, if true, could be used by Cora to counter Bernard's rejection of her explanation?

Justify

Justify

Justify

Weaken

Weaken

Weaken

Which one of the following, if true, does most to justify the conclusion above?

Strengthen

Which one of the following, if true, does NOT help to resolve the apparent discrepancy between the safety report and the city's public safety record?

Resolve X

Which one of the following, if true, enables the conclusion to be properly drawn? Which one of the following, if true, is the strongest logical counter parent P can make to parent Q's objection? Which one of the following, if true, LEAST strengthens the argument above?

Justify

Weaken

Strengthen X

Which one of the following, if true, most calls into question the claim above? Which one of the following, if true, most helps to explain the behavior of the vervet monkeys described above? Which one of the following, if true, most helps to explain the finding of the nicotine study? Which one of the following, if true, most helps to explain the puzzling fact cited above? Which one of the following, if true, most helps to explain the viewpoint of the historians described above?

Weaken

Resolve

Resolve

Resolve

Resolve

Which one of the following, if true, most helps to explain why raisins contain more iron per calorie than do grapes? Which one of the following, if true, most helps to reconcile the discrepancy indicated above?
Which one of the following, if true, most helps to reconcile the safety experts' belief with the apparently contrary evidence described above?

Resolve

Resolve

Resolve

Which one of the following, if true, most helps to resolve the apparent conflict described above? Which one of the following, if true, most helps to resolve the apparent discrepancy in the passage above? Which one of the following, if true, most seriously weakens the argument?

Resolve

Resolve

Weaken

Which one of the following, if true, most seriously weakens the reasoning above?

Weaken

Which one of the following, if true, most strengthens the argument?


Which one of the following, if true, most strongly strengthens the argument?

Strengthen

Strengthen

Which one of the following, if true, most strongly supports the statement above

Strengthen

Which one of the following, if true, most strongly supports the statement above?

Strengthen

Which one of the following, if true, most supports the argument


Which one of the following, if true, most undermines the researcher's argument? Which one of the following, if true, would most call into question the analysts' explanation of the price increase? Which one of the following, if true, would most effectively resolve the apparent paradox above? Which one of the following, if true, would most strengthen the medical doctor's argument? Which one of the following, if true, would provide the most support for the economists' assertion?

Strengthen

Weaken

Weaken

Resolve

Strengthen

Strengthen

Potrebbero piacerti anche